are you tired of the a^b vs b^a questions?

Поделиться
HTML-код
  • Опубликовано: 17 фев 2019
  • Here's a very common question that asks to compare a^b vs. b^a. We will use the calculus derivative of the function f(x)=x^(1/x) to find its maximum. It's a great practice for calculus 1 students.
    This video is inspired by my old video e^pi vs. pi^e: • e^pi vs pi^e (no calcu...
    For more calculus tutorials, check out my new channel "just calculus": / justcalculus
    💪 Support this channel, / blackpenredpen
    #calculus #blackpenredpen #mathteacher

Комментарии • 1,3 тыс.

  • @blackpenredpen
    @blackpenredpen  5 лет назад +661

    TWO THINGS!
    1. Be sure to watch 3:25 for the derivative of x^(1/x) the superman way! #KickingCalculusInItsHead #CalculusFinisher
    2. Please try the problem at 11:21 2.5^3 vs. 3^2.5 I do not know if that is even possible. So, any thoughts will be greatly appreciated!

    • @JoshuaHillerup
      @JoshuaHillerup 5 лет назад +94

      2.5 is around 0.22 smaller than e. 3 is around 0.28 larger than e. But since < e of y=x^(1/x) increases a lot faster in general that >e, and both values are still fairly far from e for that difference to dominate, 2.5^3 < 3^2.5.
      Really fuzzy reasoning there though, and it requires remembering the graph of that function in a fair amount of detail, so I don't think it's useful. Also not 100% confident that it's true.

    • @blackpenredpen
      @blackpenredpen  5 лет назад +83

      @@JoshuaHillerup But the graph isn't symmetrical about x=e, so yea...

    • @martind2520
      @martind2520 5 лет назад +162

      @@blackpenredpen
      2.5^3 = 2.5^2.5 * 2.5^0.5
      3^2.5 = 2.5^2.5 * (3/2.5)^2.5
      So the question becomes which is bigger 2.5^0.5 or (3/2.5)^2.5
      (3/2.5)^2.5 simplifies to (72sqrt(3)/125) * 2.5^0.5
      So the question becomes is 72sqrt(3)/125 bigger or less than 1? Square it and the answer is 15552/15625, which is less than 1, hence 3^2.5 is smaller than 2.5^3.

    • @JackXavierXD
      @JackXavierXD 5 лет назад +50

      2.5^3 vs 3^2.5
      (5/2)^3 vs 3^(5/2)
      125/8 vs (√3)^5
      125/8 vs 9√3
      125/72 vs √3
      1.736111... Vs 1.732...
      1.736111... > 1.732...
      (2.5^3)/9 > (3^2.5)/9
      2.5^3 > 3^2.5

    • @Hjerpower
      @Hjerpower 5 лет назад +6

      Call the first number a and the second number b, then solve for x where a^x = x^a , then if the second number is greater than x, then a^b is greater than b^a and vice versa

  • @christiantrujillo5590
    @christiantrujillo5590 3 года назад +455

    “I’m not doing 2019” proceeds to do 2020 instead
    poor guy never saw it coming.

  • @richardaversa7128
    @richardaversa7128 5 лет назад +272

    For the challenge question of "bases on opposite sides of the maximum" (a

    • @GreenMeansGOF
      @GreenMeansGOF 5 лет назад +7

      I found a solution but it still requires a calculator. Let me know what you think.
      docs.google.com/document/d/1n-dCsfetjwaku9coaFiyJlzhy7Cx10YrndfRAElLm_I/edit?usp=sharing

    • @cosimobaldi03
      @cosimobaldi03 2 года назад

      Great!

    • @lostphrases
      @lostphrases 2 года назад

      I agree.

  • @LS-Moto
    @LS-Moto 5 лет назад +394

    Nice video. I'm thinking of starting my own math channel in German once I'm better. Today I started chemo, and so far feeling all normal. Hope it stays this way for the rest of the cycles.

    • @blackpenredpen
      @blackpenredpen  5 лет назад +110

      Л.С. Мото glad to hear!! Wish everything the best for you!

    • @LS-Moto
      @LS-Moto 5 лет назад +40

      @@blackpenredpen Thank you very much bprp. Much appreciated :)

    • @jonathanmccain8646
      @jonathanmccain8646 3 года назад +11

      Did you end up starting a German-language math channel?

    • @vdun
      @vdun 3 года назад +8

      @@LS-Moto How’s it going dude?

    • @SlaveOfAllah12342
      @SlaveOfAllah12342 3 года назад +11

      Are you alive my man??

  • @erenyalcn9393
    @erenyalcn9393 5 лет назад +1505

    Poor calculus :( why did you kick him ?

  • @BriTheMathGuy
    @BriTheMathGuy 5 лет назад +198

    Not the solution that you’re looking for in general,but since the numbers are nice...
    (3^2.5)^2=3^5=243
    (2.5^3)^2=(2.5^2)(2.5^2)(2.5^2)=6.25^3>244
    Can be done by hand if you’re up to it.
    So (2.5^3)^2>(3^2.5)^2
    Implies 2.5^3>3^2.5

    • @tz233
      @tz233 5 лет назад +45

      You don't even have to do 6.25^3 by hand all the way...simply expand (6+.25)^3
      (6+.25)^3 = 6^3 + 3(6^2).25+3(6)(.25^2)+.25^3
      First term = 216, second term = 27, so sum of first two terms is 243. Sum of last two terms is obviously > 0 , so 6.25^3 >243

    • @BriTheMathGuy
      @BriTheMathGuy 5 лет назад +8

      tz very nice!

    • @skylardeslypere9909
      @skylardeslypere9909 5 лет назад +11

      Technically (2.5^3)^2 > (3^2.5)^2 implies that |2.5^3| > |3^2.5|

    • @keescanalfp5143
      @keescanalfp5143 5 лет назад +3

      @@jwl_william9276, Therefore you can't use the validity of the proof with the condition
      e < a < b.
      And just because the condition isn't fulfilled, there is reason why the question 2.5 vs. 3 could be extra interesting for being taken apart.
      The same of course with the well known
      2^4 vs. 4^2.

    • @chotusingh5340
      @chotusingh5340 5 лет назад +4

      by the way 2.5 is closer to e!! 😂

  • @dheerajlalwani4486
    @dheerajlalwani4486 5 лет назад +219

    Oh my God
    You just solved one of my greatest doubts in mathematics
    I knew it had some sensible method using calculus
    But I never tried to venture and solve it
    Thank you very much

  • @rohitchaoji
    @rohitchaoji 5 лет назад +266

    For a while, I was confused what chen lu is, but then it got clear. What made it funny is that you call it that on purpose.

    • @blackpenredpen
      @blackpenredpen  5 лет назад +72

      : ))))))
      Dr. P started it!

    • @davidwright8432
      @davidwright8432 5 лет назад +20

      His videos go very swimmingly because he does a lot on porpoise!

    • @dankie8617
      @dankie8617 5 лет назад +1

      lmao it sounds funny

    • @GammaFZ
      @GammaFZ 3 года назад +1

      blackpenredpen isn’t it?

    • @mohammedmadani7277
      @mohammedmadani7277 3 года назад

      @@blackpenredpen 😀😀😀😀😀😀

  • @nouration9685
    @nouration9685 4 года назад +193

    here's my method:
    9^10 [ ] 10^9 {ln both sides}
    note: the [ ] is where an equal or inequality sign goes
    ln (9^10) [ ] ln (10^9) {by taking the powers out of the ln}
    10 ln(9) [ ] 9 ln(10) {dividing both sides by 9 & dividing both sides by ln(9)}
    10/9 [ ] ln(10)/ln(9)
    now it's obvious the left side is bigger, because the difference between ln 10 and ln 9 is very little compared to 10 and 9, the slope for any log function, including ln, is very small after you pass the base number, which is e for ln
    well my method is more about logic than algebra I guess

    • @TheLucidDreamer12
      @TheLucidDreamer12 4 года назад +36

      Log_10 makes this even more obvious.
      Log(9^10) = 10 log(9)
      Log(9) ~ Log(10) = 1
      Log(10^9) = 9
      10 > 9

    • @speedcode5795
      @speedcode5795 4 года назад +11

      Bro the easiest way is using binomial expansion that make this question really easy to work on

    • @orlandobinungcaliii3175
      @orlandobinungcaliii3175 3 года назад +2

      @@speedcode5795 mind explaining??

    • @speedcode5795
      @speedcode5795 3 года назад +20

      @@orlandobinungcaliii3175 u can take 9^10 = (10-1)^10 and u can use permutations and combinations style of binomial expansion there will be total 11 terms and just by writting that terms u will get to know which number is biggere 9^10 or 10^9

    • @speedcode5795
      @speedcode5795 3 года назад +11

      @@orlandobinungcaliii3175 u can just search a topic called binomial expansion and u can see many vedios on that as it's taught In senior secondary classes

  • @AndDiracisHisProphet
    @AndDiracisHisProphet 5 лет назад +596

    please do the second derivative

    • @AndDiracisHisProphet
      @AndDiracisHisProphet 5 лет назад +35

      @Erik Awwad he he

    • @thebloxxer22
      @thebloxxer22 5 лет назад +19

      Me? I want to see how many derivatives it takes to reach 0.

    • @AndDiracisHisProphet
      @AndDiracisHisProphet 5 лет назад +11

      @@thebloxxer22 infinite

    • @viletomedoze5036
      @viletomedoze5036 5 лет назад +2

      @@thebloxxer22 infinte

    • @colinjava8447
      @colinjava8447 5 лет назад +15

      @@thebloxxer22 it will never reach the function f(x)=0, it will just keep getting more complicated cause it's not as simple as a polynomial

  • @ChrisAsHell
    @ChrisAsHell 5 лет назад +70

    Me in homeworks: "please dont make me do the second derivative" :v

    • @anujnautiyal8539
      @anujnautiyal8539 3 года назад

      Derivative is easy but please I don't want those integrals in my notebooks 😭

  • @calyodelphi124
    @calyodelphi124 5 лет назад +25

    This is definitely a much more rigorous mathematical proof to this type of problem, and not one I would've thought of until now. The way I solved 9^10 vs 10^9 in my head almost as quickly was this:
    1. For two positive integers x and y, such that the question asked is x^y vs. y^x, which is bigger?
    2. Take the y-th derivative of x^y, you ultimately end up with y!x.
    3. Take the x-th derivative of y^x, you ultimately end up with x!y.
    4. If you compare x and y, then whichever one is bigger will have a duplicated factor in its factorial:
    > A. If x < y, then y!x will have an x^2 term in it, whereas x!y will merely be x!y. Therefore, x^y > y^x.
    > B. But if y < x, then x!y will have a y^2 term in it, whereas y!x will just be y!x. Therefore, y^x > x^y.
    The 10-th derivative of 9^10 is 10!x9 whereas the 9-th derivative of 10^9 is 9!x10 which is just 10!. 10!x9 is strictly greater than 10! because it has an extra factor of 9 in it. Therefore, 9^10 > 10^9.
    The logic of this method is this: An n-adic function (quadratic, cubic, quartic, etc.) always increases in value faster than a linear function. Therefore, taking the respective n-th and m-th derivatives of the two functions will reveal which one is n-adic and which one is linear by virtue of their coefficients. You can conclude then that one is ultimately greater than the other.
    However, this technique fails when either x or y are not positive integers. Although Dr. Peyam has shown how to take the a-th derivative of a functionfor some positive real number a, it's a laborious process involving integration, and doesn't mix and match with the other differentiation rules at all. And afaik it's not possible to take the a-th derivative of a function for ANY negative value of a, integer OR real.
    Also, when you have two numbers that are on either side of the critical value e in x^(1/x) (or the x-th root of x), something to look for is whether a or b is less than or equal to 1. If that is the case, then the function with that (a|b) e. Therefore, 1 is another special value of the function x^(1/x). I haven't done the second derivative of it yet (and I plan to after I finish posting this comment, just for the fun of it), but I would wager that x=1 will be a point of inflection for the function because it has that special property of also being the point where the left tail of the function is permanently less than the right tail, by virtue of y=1 being the horizontal asymptote and the right tail therefore always being greater than 1. But I'm curious to see where the point of inflection is for the right tail where x > e.

  • @abj136
    @abj136 5 лет назад +60

    "Thank you for this cool t-shirt that I'm hiding behind my giant ball mic so you can't see it!"

  • @Jonasz314
    @Jonasz314 3 года назад +4

    For your follow up question, we know that the function you describe increases first then decreases. If we pick 1 < a < e < b (if a < 1, there are no solution and b^1/b will always be larger than a^1/a), we first pick a and want to find the value x for which x^(1/x) = a^(1/a) with x>e. If we have this, then we know that for b > x, b^(1/b) will be smaller than a^(1/a), and larger if e < b < x.
    The problem is to solve the general equation x^(1/x) = k. If we take the log, we get to ln(x) = h.x (with h = ln(k)) which can be solved using the Lambert W function:
    x = e^(hx) => x.e^(-hx) = 1 => -hx.e^(-hx) = -h, we apply W on both sides to get -hx = W(-h), thus x = -W(-h)/h.
    In our problem, we probably need to take the other branch of the W function, since we'd get x = a otherwise.
    If we for example plug in a = 2, this gives us x = -2 * W(-1, -ln(2)/2) / ln(2), but since W(-1, ln(2)/2) is -2ln(2) according to Wolfram Alfa, we get to the expected value x = 4.
    If you plug in a = 2.5, there's no shortcut, we get x = -2.5 / ln (2.5) * W(-1,-ln(2.5)/2.5), Wolfram Alpha gives and approximate value of 2.97029.
    You can verify that 2.5 ^ (2.5) is approximately the same as 2.97029 ^ (2.97029), within 6 decimal places.

  • @ikiyekatlananparsomen
    @ikiyekatlananparsomen 9 дней назад

    bro that video made me so excited. like, it is kind of an art and magic! thank you for those awesome videos. I have learned a lot from your videos and always kept my math passion alive.

  • @kamek80
    @kamek80 2 года назад

    This video inspired me a lot! I like your enthusiasm and smiley attitude!

  • @HarshRajAlwaysfree
    @HarshRajAlwaysfree 5 лет назад +62

    Woah that's some real superman way to do the derivative of x^1/x
    I always wished if I could do that someway without taking "ln" both sides...

    • @jeroenmampaey1183
      @jeroenmampaey1183 4 года назад +11

      you can if you have had multivariable calculus
      if you call f(u,v)=u^(1/v) and then say that u=x and v=x then there exists a "chain rule" that says df/dx=(df/du)*(du/dx)+(df/dv)*(dv/dx), du/dx and dv/dx are obviously equal to 1 and the rest you can easily work out, this rule applies for any multivariable function even with more then 2 variables

    • @MIRIYALAROHITMAHANANDICS
      @MIRIYALAROHITMAHANANDICS 3 года назад

      take a = x^1/x and apply log on both sides bruhh.

  • @QuentinStephens
    @QuentinStephens 4 года назад +45

    Even without checking the comments my first thought was to take logarithms of both sides. It is then usually trivial to demonstrate the comparative values. This works on all positive values. Maybe negative ones too. (No idea about complex numbers and quaternions.) And you don't have to take logs using base 10 or base e. Still, it was nice to be shown the general proof.

    • @paulkolodner2445
      @paulkolodner2445 2 года назад

      @@pichisnoweasel7977 If f(x) = x^(1/x), then let g(x) == ln(f(x)) = ln(x)/x. g'(x) = 1/x^2 - ln(x)/x^2 = (1-ln(x))/x^2.
      This is negative for ln(x) > 1 or x>e. The same must be true for the original f(x) because the logarithm is a monotonic function.

    • @juv7026
      @juv7026 Год назад

      @@pichisnoweasel7977 taking logarithm base 10, we need to compare:
      10log9 and 9
      it is sufficient to show log9>0.9 to see that 9^10 is greater
      log9=log(10-1)=1+log(1-1/10)>=1-1/10=0.9 (taylor's expansion) from where it is clear.

    • @GeekProdigyGuy
      @GeekProdigyGuy Год назад

      similar ineq of ln(a)/a > ln(b)/b, which is "obvious" because 1/x gets smaller much faster than ln(x) gets bigger for x>e (compare derivatives)

  • @mohghz1840
    @mohghz1840 5 лет назад +1

    I love this kind of questions, because I have watched e^pi vs pi^e video. Your videos are great man 😍

  • @lucatavianmilano
    @lucatavianmilano 4 года назад +1

    You can use the shape of the function and the equality 2^4 = 4^2 to split the problem in 4 intervals 0 < w < 2 < x < e < y < 4 < z. You get some insight for some values across e.
    Then w^y < y^w and x^z > z^x

  • @christianrodriguez823
    @christianrodriguez823 5 лет назад +9

    Very nice! Wish I had known this trick when my Calc 2 prof gave us a similar problem for homework

  • @rakhimondal5949
    @rakhimondal5949 5 лет назад +23

    Or use e^ln(9^(10)) and then solve
    Ad you did for the other video for e^π and π^e

  • @petrie911
    @petrie911 2 года назад +1

    (5/2)^3 vs 3^(5/2) is equivalent to 5^6 vs 2^6 * 3^5. You could mentally multiply this out, but it's easier to note that 3^5 = 243 < 244 = 122 * 2, and then note that 128 * 122 = 5^6 - 3^2 from the difference of squares identity. So 5^6 > 2^6 * 3^5, and thus (5/2)^3 > 3^(5/2).
    Of course, this solution isn't general, but I suspect there isn't one for the case a < e < b.

  • @nahuelporta1648
    @nahuelporta1648 4 года назад

    I may be tired of this kind of questions but i never will be of this kind of explanations. Great video! Keep it up

  • @gregorymorse8423
    @gregorymorse8423 4 года назад +5

    The second derivative yields slope on each side of e. This could be used to compare rate of change on those sides and give a criterion for the a and b on opposite sides though the formula will be more complicated than whichever is nearer to e.

  • @koenth2359
    @koenth2359 2 года назад +6

    Cool! For the case in the question, I just noticed that the first two terms of the binomial expansion of (10-1)^10 cancel out and the third (dominating) term 90/2*10^8 is big enough to easily beat both the fourth term -720/6 * 10^7 and the other side of the equation, 10^9.

  • @joaopedrolealmaran77
    @joaopedrolealmaran77 5 лет назад +11

    Hey Steve, i have to say that i just saw your channel and i already love it. You really do a great job and i love the way you are fascinated by math. Just gained another fan :)

    • @blackpenredpen
      @blackpenredpen  5 лет назад +5

      Joao Pedro Leal Maran thank you Joao!!! I am very glad to hear it!!

  • @theimmux3034
    @theimmux3034 4 года назад

    One of your coolest videos imo, really nice

  • @MrBoubource
    @MrBoubource 5 лет назад +4

    Let's just use a calculator.
    Let a < e < b
    Is it easy to find the unique c > e such that c^(1/c) = a^(1/a)? Because then we could just use our first theorem.

    • @GreenMeansGOF
      @GreenMeansGOF 5 лет назад +3

      I wanted to share something I discovered based on your suggestion. Suppose we are comparing 2^(1/2) to 5^(1/5). Using the Lambert W function, I figured out that 2^(1/2) = 4^(1/4), which is easy to see. We know 4^(1/4) is greater than 5^(1/5) so 2^(1/2) is also greater. I think that we may have finally geeralized this problem. What do you think, blackpenredpen?

    • @MrBoubource
      @MrBoubource 5 лет назад

      @@GreenMeansGOF so the idea is to get to the form x^(1/x) vs y^(1/y) using the Lambert W function, with x and y greater than e? I was missing Lambert if this indeed works!

    • @GreenMeansGOF
      @GreenMeansGOF 5 лет назад +2

      The idea is to get two numbers that are either both greater than e or both less than e. Note that 2

  • @ArnabAnimeshDas
    @ArnabAnimeshDas 4 года назад +4

    However, if 0

  • @jonathansegal1241
    @jonathansegal1241 2 года назад

    Excellent video. I saw that you did a proof involving linear algebra and the transpose of The product of two matrices. You’re awesome. That was an excellent video. You are very very clear

  • @collinsbulaya3077
    @collinsbulaya3077 3 года назад

    I enjoy your maths. You made me understand well trigonometric identities and proofs.

  • @MCredstoningnstuff
    @MCredstoningnstuff 5 лет назад +5

    This reminds me about some math a did a couple years ago with tetration! I found that when you infinitely tetrate x, you get a function of x=y^(1/y). But only for 0

    • @MCredstoningnstuff
      @MCredstoningnstuff 5 лет назад +1

      I actually have a lot more on this but it won't fit in a youtube comment.

    • @blackpenredpen
      @blackpenredpen  5 лет назад

      @@MCredstoningnstuff : )))))))

    • @angelmendez-rivera351
      @angelmendez-rivera351 5 лет назад +1

      @@MCredstoningnstuff A little inaccurate. Rather, infinite tetration is defined everywhere by analytic continuation, but the sequence of partial tetrations only converges if 1/(e^e) < x < e^(1/e).

    • @MCredstoningnstuff
      @MCredstoningnstuff 5 лет назад +1

      @@angelmendez-rivera351 you're right! It does follow this function on the lower part of the bifurcation after 1/(e^e) but the whole thing isn't defined as a function. I actually don't know what function the upper half follows. Thanks for the correction

  • @GreenMeansGOF
    @GreenMeansGOF 5 лет назад +52

    2.5^3 vs 3^2.5
    (squaring)
    2.5^6 vs 3^5
    (multyplying by 2^6)
    5^6 vs 2^6*3^5
    5^6 vs 2*6^5
    Clearly the left hand side is greater. My issue with this is that this technique likely will not work in general.

    • @blackpenredpen
      @blackpenredpen  5 лет назад +16

      GreenMeansGO
      Yea... that’s my main question too. For example (sqrt3)^pi vs pi^sqrt(3)

    • @ffggddss
      @ffggddss 5 лет назад +24

      @ GreenMeansGO: Clearly?
      5⁶ = 15625
      6⁵ = 7776; 2·6⁵ = 15552
      . . a pretty close shave; about ½% difference.
      PS: Kudos for getting down to integers! I stopped short of that in my own answer.
      Fred

    • @angelmendez-rivera351
      @angelmendez-rivera351 5 лет назад +2

      @@blackpenredpen I was writing a comment in which I developed a general theorem to compare a^b and b^a, but my phone died in the process :( I will rewrite it again, so comparing pi^sqrt(3) and sqrt(3)^pi will not be so much of a problem any longer.

    • @GreenMeansGOF
      @GreenMeansGOF 5 лет назад

      Hi Fred. You know, I honestly do not know how I made the final conclusion. My analysis is incomplete. I lack a justification as to why the left hand side is greater. Regardless, we need a more general technique. Also, thank you for your acknowledgement of the integers. This worked because our numbers were positive and rational. Perhaps similar results can be derived for algebraic numbers. I want to think about this more.

    • @ffggddss
      @ffggddss 5 лет назад +2

      @@GreenMeansGOF Well, you were actually correct in the end :-)
      As far as a more general technique, I'm not sure there is one.
      Sounds like maybe Angel M-R may be on the trail to something, though, so let's stay tuned....
      Fred

  • @johnabreuolivo8119
    @johnabreuolivo8119 3 года назад +1

    Thank you for the general case. For the case involving 10 and 9, considering 'decimal log' and the fact that 'log' is an increasing function::
    It's known from HS log(3)=0.4771... --> 20*log(3) > 20*0.477 --> log(9^10) > 9 = log(10^9). --> 9^19 > 10^9.

  • @Tinybabyfishy
    @Tinybabyfishy 5 лет назад +2

    We can create another bound where it becomes easy again if we use the horizontal asymptote y=1. Choose any value a such that f(a) < 1 and we know it can never be greater than f(b) if b >= e

  • @zwz.zdenek
    @zwz.zdenek 4 года назад +3

    2.5 is going to be tough, but since most people will probably just use 2 as one of the numbers, it's useful to know that the graph has the same value at 4 allowing you to still use this method. 3^2 is "closer" to e than 2^3 because 2 is as "far" (vertically) as 4.

  • @carstenmeyer7786
    @carstenmeyer7786 Год назад +7

    With *a < e < b* there is no simple rule, since all cases may occur. Counterexamples:
    *2^3 < 3^2 2^4 = 4^2 2^5 > 5^2*
    Concerning the challenge: Define the function *f(x) = x^(1/2)* increasing for *x > 0* and notice
    *3^(2.5) / (2.5)^3 = f( 3^5 / (2.5)^6 ) = f( 2 * 6^5 / 5^6 ) = f( 15552 / 15625 ) =: X*
    Since *f* maps *(0; 1) -> (0; 1),* the result *X* lies in *(0; 1),* leading to *3^(2.5) < (2.5)^3*

  • @dimaignatiev6370
    @dimaignatiev6370 4 года назад +1

    That's really really cool,man!

  • @jjeherrera
    @jjeherrera 5 лет назад +2

    Regarding the derivative y'=x^(1/x), there's a faster way: take y=exp{ln(x^(1/x))}. Then y'=[exp{ln(x^(1/x))}][ln(x^(1/x)]'=x^(1/x)[(1/x^2)-(1/x^2)lnx].
    Nice explanation, as usual!

  • @kostantinos2297
    @kostantinos2297 5 лет назад +5

    Wouldn't it be easier to take the ln on both sides of y=x^(1/x), bring the 1/x in front, and then take the derivative? Although I believe it is essentially the same thing, it would make more sense to me.

  • @onkargangane2797
    @onkargangane2797 5 лет назад +10

    Such problems used to haunt me like anything. Thanks for this brilliant approach!!!!

  • @elizabeth8720
    @elizabeth8720 5 лет назад

    You can also use the tangent line to y=ln(x) (which is concave down) at x=9 to get ln(10)

  • @VerSalieri
    @VerSalieri 5 лет назад

    I remember Doing it using the variations of the function lnx/x.
    Edit: this is a decreasing function for all x>e.. (in fact (e,1/e) is an absolute maximum)
    then for e lnb/b then blna > alnb, or ln(a^b)>ln(b^a), which finally gives a^b > b^a.

  • @himanshugupta4395
    @himanshugupta4395 5 лет назад +4

    Great you always bring something unique :)

  • @pushkarpriyachand6290
    @pushkarpriyachand6290 4 года назад +4

    Loved how you worked with the differentiation part! Is there a proof for this?

    • @GeekProdigyGuy
      @GeekProdigyGuy Год назад

      he treated it as a multivariable function where the base and power are both variables; the total derivative is equal to the sum of the partial derivatives

  • @sanauj15
    @sanauj15 5 лет назад

    You can multiply 2.5 by itself by hand to get 2.5^3, then rewrite 3^2.5 as 3^(5/2) which is equal to the square root of 243 then use either newton's method or linear approximation to get answer.

  • @stefankourou252
    @stefankourou252 3 года назад +1

    Very nice video! I would like to ask though... is it safe to claim that if 0

    • @giobbo98
      @giobbo98 2 года назад

      Yes, a^b is smaller than b^a with 0

  • @slowedreverb6819
    @slowedreverb6819 2 года назад +3

    *Cries in bluepen*

  • @doctorlazarus8854
    @doctorlazarus8854 5 лет назад +13

    I love Chan luuu!
    *Gives a flying kiss*

  • @vijaykrishnan7797
    @vijaykrishnan7797 3 года назад

    Love the way you teach math,enjoyable

  • @abidhossain8074
    @abidhossain8074 4 года назад

    Can we do this
    // when (a>e and be and ae and b

  • @secretsquirrel4375
    @secretsquirrel4375 5 лет назад +5

    What is the curvature of the graph of y=x^(1/x)? (:

  • @pseudo_goose
    @pseudo_goose 4 года назад +8

    Me: but what if ...
    1 minute later - 11:02 bprp: *_I KNOW_*

  • @VivekSharma-vv3dd
    @VivekSharma-vv3dd 3 года назад

    Your way of teaching is awesome also u help me to memorise chp. Monotonocity by this ques as I have done this type of ques during me jee prep .

  • @Kori114
    @Kori114 5 лет назад

    These videos are so fucking satisfying! I love this!!!! YES!

  • @JoshuaHillerup
    @JoshuaHillerup 5 лет назад +14

    At first I forgot that they have to be gte e, thought "hey, 8 < 9", and then guessed wrong.

  • @muskyoxes
    @muskyoxes 3 года назад +12

    I keep forgetting the order so it helps me to put in outrageous numbers, like 3^1000 vs 1000^3. Obviously the former is greater.

    • @gabrieljohnson6304
      @gabrieljohnson6304 3 года назад +1

      @Irwan Gunardi no, 2 is smaller than e, and 8 vs 9 literally gives you the opposite of what this video is saying.

    • @aaykat6078
      @aaykat6078 3 года назад +1

      @@gabrieljohnson6304 point is you instantly solve 2³ or 3², and can know the answer

    • @gabrieljohnson6304
      @gabrieljohnson6304 3 года назад +2

      @@aaykat6078 helpful for solving that specific question, not helpful for remembering which would be bigger when both bases are larger than e

    • @benzienugent2010
      @benzienugent2010 3 года назад

      @@aaykat6078 lol

  • @Drummer4774
    @Drummer4774 3 года назад

    I think i get one extra year of life understanding that. Finally someone explain it clearly

  • @khalidibnemasoodkhalid101
    @khalidibnemasoodkhalid101 4 года назад

    1. how will you compare if you have a < e < b ?
    2. I really like your videos. you are awesome !!!

  • @almightyhydra
    @almightyhydra 5 лет назад +9

    Why did you add the two parts of the derivative? It's not a product rule situation is it?

    • @Apollorion
      @Apollorion 10 дней назад

      It actually is: y=f(x)^g(x) = e^(g(x)*ln(f(x)))
      Do you see that product in the exponent?

  • @sanjaybhandarkar5779
    @sanjaybhandarkar5779 5 лет назад +25

    I think 2.5^3 is is bigger coz 2.5 is closer to e than 3

    • @SlipperyTeeth
      @SlipperyTeeth 5 лет назад +34

      0^1000 or 1000^0. Which one's bigger?

    • @thehen101
      @thehen101 5 лет назад +5

      @@SlipperyTeeth 1000^0 bigger

    • @alephnull4044
      @alephnull4044 5 лет назад +11

      I don't think it works like that? You're assuming symmetry of the graph of log(x)/x around x=e which is obviously not correct.

    • @alephnull4044
      @alephnull4044 5 лет назад +1

      But considering the second derivative maybe such an argument can be made to work, but not the other way round.

    • @kinyutaka
      @kinyutaka 5 лет назад

      @@SlipperyTeeth 0^1000 is 0, 1000^0 is 1

  • @567secret
    @567secret 3 года назад +2

    When considering functions like y = x^f(x) you may also take the natural logarithm of both sides and differentiate implicitly.

  • @James-gd3sp
    @James-gd3sp 3 года назад

    You could make a rule for the left of e too. Calculate where y=1 on the left hand side and you know from the bounds [0,#) u [e,inf). Now there will still be a gap of known so I'm not sure on that bit.

  • @pranshusrivastava8353
    @pranshusrivastava8353 5 лет назад +4

    Wouldn't differentiation using implicit differentiation also be easy?

  • @lakshyadua1373
    @lakshyadua1373 3 года назад +6

    He and calculus are like BFF.....who personally beats him and publicly promotes his friend.
    LOL!🤣

  • @amarpratap011
    @amarpratap011 4 года назад

    Case 1 and Case 2 - when both a and b are greater than e, then the exponent dominates regardless.
    Case 3 and Case 4 - when both a and b are less than e, then the base dominates regardless.
    Case 5 - When b

  • @blake121666
    @blake121666 2 года назад

    Looking at the x^(1/x) graph you put up there, I think it might be illustrative to find values to the left of e that equal values to the right of e. So you should probably shift the independent variable by e and use the generalized binomial theorem for the new independent variables (e - x) (for a) and (x -e) (for b). The points at which the one equals the other then gives the same situation as before - the ranges where one is larger than the other. You'd get points to the left of the first point PLUS to the right of the second point means the "a" part is greater - and in between those points the "b" part is greater.
    Starting at e, moving to the left will give a's to the left PLUS a domain of values to the far right - which will become increasingly smaller the further you go - and stopping at a=1 (as another commenter has noticed - for the asymptote value)

  • @HK-cq6yf
    @HK-cq6yf 5 лет назад +5

    What kind of whiteboard is that? I don't see any marker residue or ghosting at all. How do you keep it so clean?

    • @blackpenredpen
      @blackpenredpen  5 лет назад +7

      Peter Kim well... I spent a lot of time erasing before filming....

    • @HK-cq6yf
      @HK-cq6yf 5 лет назад +3

      @@blackpenredpen I see...any tips for erasing so well?

    • @yusufat1
      @yusufat1 5 лет назад +2

      wipe the board with a cloth soaked in ethyl alcohol (ethanol). Works everytime.

    • @keescanalfp5143
      @keescanalfp5143 5 лет назад +2

      @@HK-cq6yf, And use the right marker pens. And - quite cheap tip - don't enjoy all too long but be quick with erasing what you've just filmed. Never let it overnight…

  • @tryphonunzouave8384
    @tryphonunzouave8384 5 лет назад +3

    In a math exam would you have to demonstrate why you pick one or the other, or would you just pick one and say "that one is bigger" ?

    • @tryphonunzouave8384
      @tryphonunzouave8384 5 лет назад +1

      Also you bring me a lot of joy through the day thanks for you everlasting happiness

  • @christianfunintuscany1147
    @christianfunintuscany1147 4 года назад +1

    (2.5)^3 = 125/8 = 15.625
    3^(2.5) = 9 sqr(3) = 15.588
    for small numbers it is easy to evaluate the quantities ... your analysis is very useful to answer the question when big numbers are considered

    • @fipaan
      @fipaan 2 года назад +1

      (2.5)^3 and 3^(2.5)
      ((2.5)^3)^2 and (3^(2.5))^2
      (2.5)^(3*2) and 3^(2.5*2)
      2.5^6 and 3^5
      Next calculations easy
      (Not must need to know square of 3)

  • @ashishmohan8777
    @ashishmohan8777 4 года назад

    Sir ,you have mind blowing way of teaching .....👌👌👌👌👌

  • @helo3827
    @helo3827 3 года назад +3

    he said: don't use a calculator,
    me: uses a calculator.

  • @yath3681
    @yath3681 5 лет назад +5

    Use binomial theorem.. write 9^10 as (10-1)^10 then compare the two

  • @mementomori7160
    @mementomori7160 5 лет назад +2

    If numbers are on the opposite side in some cases there is a way.
    1. We have numbers a and b, that 0 < a

    • @mementomori7160
      @mementomori7160 5 лет назад +1

      @CogitoErgoCogitoSum Then they are on the same side of the graph, so the one closer to e is our winner, I'm talking when they aren't on the same side "If numbers are on the opposite side"

  • @tarungupta3516
    @tarungupta3516 4 года назад +2

    Wow 😮 what a trick. It definitely helped me
    ❤❤❤Love from India🇮

  • @Gatchet
    @Gatchet 5 лет назад +10

    I mean come on bprp you're basically begging for us to ask you to do the second derivative!

    • @takix2007
      @takix2007 3 года назад

      Moreover it's not that difficult, [x^(1/x)]/(x^2) = x^(1/x-2), which makes it "easy" given we already have [x^(1/x)]' 😜

  • @aswrestling9920
    @aswrestling9920 5 лет назад +30

    can binomial theorem be used to solve this? (Can write 9^10 as (10-1)^10 and then expand it..?)

    • @farooq8fox
      @farooq8fox 5 лет назад +13

      AS Wrestling Isnt that just more complicated?

    • @angelmendez-rivera351
      @angelmendez-rivera351 5 лет назад +12

      You can solve it using the binomial theorem, but as someone else said, that is noteably more complicated.

    • @aswrestling9920
      @aswrestling9920 5 лет назад +4

      @@angelmendez-rivera351 Tbh, at times I find calculations easy for binomial theorem (Like till the power 7-8) I haven't even tried 10 yet.. but i guess it's manageable..

    • @blackpenredpen
      @blackpenredpen  5 лет назад +19

      Yes it can!

    • @dolevgo8535
      @dolevgo8535 5 лет назад +2

      @@aswrestling9920 i mean.. if you're going that way wouldn't it be easier to just multiply 9 by itself 10 times?

  • @luisrosano3510
    @luisrosano3510 5 лет назад

    Very nice explanation!!!

  • @scepticusverisimillimenonm8450
    @scepticusverisimillimenonm8450 4 года назад +1

    Hmm. The equation x^(1/x)=a has two solutions for 1

  • @RITESHKUMAR-fq6js
    @RITESHKUMAR-fq6js 5 лет назад +7

    Please give lectures on real analysis

  • @13579YOOTUBE
    @13579YOOTUBE 5 лет назад +3

    I liked the inequality and its maxima

  • @tubax926
    @tubax926 3 года назад

    Watching this at 3 am and not understanding the reason behind half the stuff you did but I still understood the concept. I'll try this myself when I'm free lol

  • @diego_sabbagh
    @diego_sabbagh 4 года назад

    11:44 actually we have an horizontal asymptote at 1, so if a is on the "left side" (meaning ae) but a^(1/a)

    • @hach1koko
      @hach1koko 4 года назад

      True but if a1 then it's pretty clear that a^b1 so you can see that directly

  • @jp0678
    @jp0678 5 лет назад +6

    But why do you have blue pen also? (with your channel name only having black and red)
    just curious :P

  • @geethaudupa8930
    @geethaudupa8930 5 лет назад +8

    Hey bprp can you solve
    x^(x+1)=(x+1)^x ?
    I'm stuck with this

    • @user-hg2di5ye4r
      @user-hg2di5ye4r 5 лет назад +1

      hey, i've tried to solve it by hand, but i couldn't. It's quite obvious, that x shouldn't be greater(or equal) than e, otherwise we have a=x and b=x+1(e

    • @geethaudupa8930
      @geethaudupa8930 5 лет назад

      @@user-hg2di5ye4r thanks man:)

    • @angelmendez-rivera351
      @angelmendez-rivera351 4 года назад

      The second Foias constant is, in a sense, *defined* to be the real solution to x^(x + 1) = (x + 1)^x. There is no closed-form expression for it.

  • @davidwright8432
    @davidwright8432 5 лет назад +2

    Thanks again for lots of fun, and intriguing math! You keep my ancient brain chugging over on at least one of its many cylinders. Two, on a good day - like this!

  • @kajlundgren7977
    @kajlundgren7977 2 года назад

    if abs(e-a) almost = abs(e-b) then and 0

  • @rituchandra6325
    @rituchandra6325 5 лет назад +4

    can you explain why the derivative in the superman way works??? i tried it for a general y = f(x)^g(x) and it works for it! how?!

    • @philosandsofost8642
      @philosandsofost8642 5 лет назад

      I don't know what the "superman method" is, but, maybe you can figure that out by using implicit differentiation and look out for a pattern

    • @rituchandra6325
      @rituchandra6325 5 лет назад

      @@philosandsofost8642 watch the video... he explains it at 3:25

    • @angelmendez-rivera351
      @angelmendez-rivera351 4 года назад

      f(x)^g(x) = exp{g(x)·log[f(x)]}, hence the derivative of f(x)^g(x) can be evaluated via the chain rule to be exp{g(x)·log[f(x)]} multiplied by the derivative of g(x)·log[f(x)], which is given by g'(x)·log[f(x)] + g(x)·f'(x)/f(x). Therefore, the derivative of f(x)^g(x) is equal to exp{g(x)·log[f(x)]}·{g'(x)·log[f(x)] + g(x)·f'(x)/f(x)}. Since exp{g(x)·log[f(x)]} = f(x)^g(x), this implies that the derivative of f(x)^g(x) is equal to f(x)^g(x)·{g'(x)·log[f(x)] + g(x)·f'(x)/f(x)} = f(x)^g(x)·log[f(x)]·g'(x) + g(x)·f(x)^[g(x) - 1]·f'(x).

  • @GRBtutorials
    @GRBtutorials 4 года назад +8

    Interesting. I arrived to the same conclusion instinctively, because 2⁴ = 4² and exponential functions rise faster than polynomial functions, so for numbers greater than 2/4 it was the most likely answer.
    PS: what’s going on with the first part of your derivative? Isn’t it just chain rule?

    • @Banzybanz
      @Banzybanz 2 года назад +2

      Other than a = 1, 2³ < 3² and 2⁴ = 4² are the only integer counterexamples for a < e.

  • @msolec2000
    @msolec2000 5 лет назад +4

    What in tarnation did you do with derivation rules???

  • @kaustubhj24
    @kaustubhj24 4 года назад +2

    It's easier to do it by log, as log is a positively increasing function the log of one number will be higher than the other respectively which boils down to comparing 20*ln2 ans 9*ln10 clearly 1st one is bigger.

  • @Kernel15
    @Kernel15 5 лет назад +9

    I did the lazy estimate method.
    I used 10^10 to divide both.
    10^10 / 10^9 = 10
    10^10 / 9^10 = (10/9)^10
    (10/9)^10 = (10/9)^9 * 10/9
    (10/9)^10 = [(1000/729)^3 * 10/9] < [2^3 * 1.25] (which is 10)
    Hence 9^10 is larger.

  • @hammer.11011
    @hammer.11011 5 лет назад +3

    I somehow remember the values of log3 and log5 (base 10) (crammed for an exam) and that gives me
    3log2.5 > 2.5log3
    Edit: upto 5 decimal places

  • @mcoolgamer12
    @mcoolgamer12 3 года назад +1

    Note: I have no idea what e means, thus here is my idea 2³

  • @1412-kaito
    @1412-kaito 5 лет назад

    Hey I believe that you said that on the right side of the graph forms an acentonde at Y = 1 , so on left side to get a value of 1 or less than 1 the number it self has to be 1 or less than 1 , right ?
    So by what you said doesn't it means that value on left side will always be greater as long as on left side the base is 1 or less than 1 ?
    Given that the number is not really really close to e as near e^(1/e) the values will varry.

  • @paytonrichards784
    @paytonrichards784 5 лет назад +3

    If a is less than 1 and b is greater than e then it's pretty easy.

  • @damianbla4469
    @damianbla4469 2 года назад +7

    03:20 WOW! Taking the derivative of x^(1/x) using "power rule plus exponential rule".
    I am wondering if it works for all such functions, for example for sin(x)^ln(x).

    • @blackpenredpen
      @blackpenredpen  2 года назад +1

      Maybe with chain rule. Not sure tho

    • @LegendOfRoGamers
      @LegendOfRoGamers 2 года назад +1

      You can also use a property: a^b = e^(b*ln(a)) and then it's a bit easier to get the derjvative if you ever have doubts. My HS math teacher showed us this way when demonstrating this derivative

    • @xavidoor
      @xavidoor 2 года назад +1

      Yes, the method works for all functions of the form f(x)^g(x) if you apply the chain rule to the función which you are considering variable in each steñ of the derivation. You can check ir by deriving the general expression y=f(x) ^ g(x) using the traditional method of applying ln to both sides, or using the "power rule plus exponential rule". You will arrive to two expressions that can be easily shown to be equal.

  • @dackid2831
    @dackid2831 4 года назад +1

    I have a theory on the final question. Based on your graph, it seems as if there is a value that the limit is approaching, that isn't 0 (Granted, you sketched the graphs). If the graph of x^1/x looks relatively like that, then we can determine the value based on the limit. If the number left of e is lower than that limit, than the number to the right will be bigger.
    Now if the left side is above the limit, than that all depends how far away the each function is from the limit.
    This is only based on the graph, and therefore might be total bogus... but if your graph holds up, I think this would work!

  • @VerSalieri
    @VerSalieri 3 года назад

    I love e and ln...once asked my students to graph the function y=x^(1/lnx) which is almost a horizontal line...just almost.